Matrix Exponential Homework: Eigenvalues & Eigenvectors

Click For Summary
The discussion focuses on proving that if λ and V are an eigenvalue and eigenvector of matrix A, then e^A V = e^λ V. Participants suggest starting from the relationship A V = λ V and exploring higher powers of A acting on V, such as A² V and A³ V. There is also a mention of the convergence of the power series for matrix exponentials, indicating a deeper mathematical inquiry. The thread emphasizes the need for a structured approach to the problem, as the original poster has not demonstrated sufficient effort in their attempt. Overall, the conversation highlights the importance of understanding eigenvalues and eigenvectors in the context of matrix exponentials.
BearY
Messages
53
Reaction score
8

Homework Statement


Show that if ##λ##and ##V ## are a pair of eigenvalue and eigenvector for matrix A, $$e^Av=e^λv$$

Homework Equations


##e^A=\sum\limits_{n=0}^\infty\frac{1}{n!}A^n##

The Attempt at a Solution


I don't know where to start.
 
Last edited:
Physics news on Phys.org
I'd use a lower case ##v## or better ##\mathbf v## here. (I've never seen capitalized vectors?)
BearY said:

Homework Equations


##e^A=\sum\limits_{n=0}^\infty\frac{1}{n!}A^n##
The power series is quite instructive. Each side is some matrix that is a function of ##A##. What happens if you multliply each side by an eigenvector of A? The idea is there are a lot of moving parts here... if you can find a fixed point maybe it isn't so hard.
- - -
btw, do you know why the series is convergent for matrices? This is a bit subtler of a question.
 
  • Like
Likes BearY
BearY said:

Homework Statement


Show that if ##λ##and ##V ## are a pair of eigenvalue and eigenvector for matrix A, $$e^AV=e^λV$$

Homework Equations


##e^A=\sum\limits_{n=0}^\infty\frac{1}{n!}A^n##

The Attempt at a Solution


I don't know where to start.

Start with ##A V = \lambda V##. What are ##A^2 V##, ##A^3 V##, etc.?
 
  • Like
Likes BearY
Thread locked as the OP has shown no effort.
 
Question: A clock's minute hand has length 4 and its hour hand has length 3. What is the distance between the tips at the moment when it is increasing most rapidly?(Putnam Exam Question) Answer: Making assumption that both the hands moves at constant angular velocities, the answer is ## \sqrt{7} .## But don't you think this assumption is somewhat doubtful and wrong?

Similar threads

  • · Replies 5 ·
Replies
5
Views
2K
  • · Replies 2 ·
Replies
2
Views
2K
Replies
8
Views
3K
  • · Replies 8 ·
Replies
8
Views
2K
  • · Replies 6 ·
Replies
6
Views
2K
  • · Replies 5 ·
Replies
5
Views
14K
  • · Replies 19 ·
Replies
19
Views
4K
  • · Replies 2 ·
Replies
2
Views
2K
  • · Replies 4 ·
Replies
4
Views
3K
  • · Replies 5 ·
Replies
5
Views
2K